9,441 results
Question

Question 2688

A preterm neonate has been treated in the neonatal ICU since birth and is now 10 days old. They are intubated and ventilated, and while there was an initial good response to surfactant treatment, their oxygen requirement has increased over the last 12 hours.

A chest x-ray is performed and demonstrates linear lucency extending from the hila towards the periphery of the film. What is the likely diagnosis?

Question

Question 2028

Which of the following describe features seen in sarcoidosis?

Question

Question 29

All of the following could be an underlying predisposing factor for this patient, EXCEPT...

Question

Question 1631

A 60-year-old man presents in the emergency department with complaints of shortness of breath. His chest x-ray is presented which shows all of the following features of congestive cardiac failure EXCEPT...

Question

Question 2666

A 35-year-old male presents with unilateral hearing loss and otorrhoea. Which of the following is the most likely diagnosis from this image?

Question

Question 32

Which valve separates the right atrium from the right ventricle?

Question

Question 1911

All the following situations may be considered as risk factors for intracranial atherosclerotic disease (ICAD) EXCEPT... 

Question

Question 2545

Chest radiograph of a young adult. The outline of what normal structure is indicated by the arrows?

Question

Question 2567

At which level is the oesophagus narrowest?

Question

Question 2294

What 2 findings besides volume loss and a round shape are characteristic of round atelectasis?

Question

Question 244

What is the most common site of traumatic thoracic aortic injury?

Question

Question 247

Which of the following is the most common cause of a lobar pneumonia? 

Question

Question 448

A 20-year-old male, previously well, presents with a four-day history of fevers, and a chest x-ray is performed. What is the most likely diagnosis? 

Question

Question 387

All of the following are pneumoconiosis EXCEPT...

Question

Question 388

Which of the following is LEAST likely to cause a lobar pneumonia? 

Question

Question 2609

Which of the following statements regarding pulmonary haematoma is NOT true? Pulmonary haematomas...

Question

Question 1495

A 70-year-old female presents with hand pain. A PA hand and wrist x-ray is shown. What is the most likely diagnosis?

Question

Question 980

A Gerbode defect is a communication between the...

Question

Question 1217

Sagittal PD weighted MRI image of the shoulder. What structure has been arrowed? 

Question

Question 1313

Which one of these conditions is associated with extramedullary haematopoiesis?

Updating… Please wait.

 Unable to process the form. Check for errors and try again.

 Thank you for updating your details.